A row of 1000 integers

A row contains 1000 integers

The second row is formed by writing under each integer, the number of times it occurs in the first row

The third row is now constructed by writing under each number in the 2nd row, the number of times it occurs in the 2nd row

This is process is continued

Prove that at some point, one row becomes identical to the next.

[I could give the source too since the author has given a very cryptic clue in place of a solution. Its from Chapter 1 of Problem Solving Strategies by Arthur Engel]

26 Answers

11
Devil ·

Prophet sir, pls wait for some-time before giving the soln.....me trying it.

11
Devil ·

Yup, prophet sir, this was a gr8 "non-routine" prob, even though i cud get nowhere with this, still I enjoyed trying it, waiting for more from ur side.

1
Kalyan Pilla ·

Yeah though it is more of a puzzle, it took in a lot of logic to solve and was a very good one indeed

341
Hari Shankar ·

yeah. its more of a puzzle. that's what makes it sort of frustrating because there are no expressions to manipulate, no derivatives or integrals to take! The approach to be used is so open-ended.

Unless you play around with some example sequences and make observations you just dont know how to proceed.

But the result obtained is so dramatic, I felt it was worth sparing some thought for it :D

9
Celestine preetham ·

so this thread ends here :)

but i shud say this is a good Q but not of olympiad level

341
Hari Shankar ·

Ok the main points to be put together are

1. From 2nd row onwards a number k occurs n times where n is a multiple of k (celestine has already said this)

2. This means that the numbers that appear underneath k in the succeeding row are never less than k.
(Kaymant Sir)

3. If nk represents the number of times the number k occurs then Σnk = 1000 for k appearing in the row.(Celestine)

4. We have said that nk = mkk. The claim is that the rows start to become identical when mk=1 for all k appearing in the row.(Celestine said this, but this has to be spelled out in this way so that we can conclude that:)

This is bound to happen as the numbers in succeeding rows keep increasing. All The coefficients gotta turn out to be 1 at some stage and remain so thereafter.

1
Kalyan Pilla ·

say, the numbers are a,c,b,b,c,a,s,g,b,b,f,g,d,s,s,a

the next row will be 3,2,4,4,2,3,3,2,4,4,1,2,1,3,3,3

the third row will be 6,4,4,4,4,6,6,4,4,4,2,4,2,6,6,6

4th row 6,8,8,8,8,6,6,8,8,8,2,8,2,6,6,6

Fifth row is the same as the fourth one.......

If a number repeats k times, 'k' is written k times in the next row,

while, if n numbers repeat k times, 'k' is written nk times in the next row

Further in the next row 'nk' is written nk times in the next row, whatever the positions be

Similarly other numbers come to the third row.

Now in the third row if nk=mp=~~, (mp, if m numbers repeat k times in the first row)and if there x such equivalent conditions,

the fourth row contains 'nk', xnk times

and the fifth row has 'xnk' written xnk times.

This process continues until there are no such equivalence conditions.

Say, these equivalence conditions occur q times,

then (2q+3)th and (2q+4)th rows are identical.

I dont know if the explanation is clear, but that is what I could infer after trying many numbers in a program created on the basis of the given conditions!!![3][4]

[339]

9
Celestine preetham ·

sir can u post ur argument why #10 or # 16

are not fully convincing solutions ?

they appear fully convincing to me :P

341
Hari Shankar ·

Yeah, non-decreasing is ok. But from there is a short leap to be taken to land at the solution. Meanwhile celestine is online and come with a solution I hope.

66
kaymant ·

I think even in the situation mentioned in #18 my arguments remain valid. Each column will be non-decreasing even in that case. But still let me see.

341
Hari Shankar ·

Actually this argument too will not lead us to the solution.

What could happen is that a certain number b appears b times. But another number a also appears b times. This means a number b does not necessarily remain constant across rows after if it has appeared b times in a particular row.

341
Hari Shankar ·

A small detail is missing. If that is supplied, then the problem will be convincingly closed.

Hint: (already there in cele's post) under what conditions does a row become identical to the previous?

66
kaymant ·

I used the idea of the hint given in the book itself and came up with the following:
Suppose that in the n\mathrm{th} row some number a occupies exactly b places. Then in the (n+1)\mathrm{th} row these b positions are occupied by the number b. Hence for each n\geq 2 we can conclude that:

The number of occurrences of an integer a in the n\mathrm{th} row is an integral multiple of a, say ka where k is the number of those entries which appeared exactly a times in the (n-1)\mathrm{th} row.

As such from the n\mathrm{th} row onwards, we have beneath the entries a an integers ka\geq a. Hence, the sequence of the integers in each column beginning with the second row is an non-decreasing sequence of numbers which is bounded by 1000 above. As such in each of the 1000 columns the same number is going to be repeated from a certain row onwards. Accordingly, sooner or later we shall come across a row starting from which all the subsequent rows are going to be identical.

9
Celestine preetham ·

well i cud infer that a particular integer x shud be repeated x times for it to be feasible

trying to solve

62
Lokesh Verma ·

Will it suffice that there are a finite number of n digit numbers?

9
Celestine preetham ·

ive indeed proven mathematically only sir !

though im lost for words on how to express it

(i shud admit im quite busy with admission process pls excuse me )

341
Hari Shankar ·

That is not the crux observation that will solve the problem.

Also it is possible to prove it mathematically. So do try some more.

9
Celestine preetham ·

eg

1 1 1 1 1 2 2 2 2 3 3 3 3 randomly distinct nos

next row bcoms

5 5 5 5 5 4 4 4 4 4 4 4 4 randomly distinct nos

so the next step is obvious

:D

9
Celestine preetham ·

the solution is such that i need to express in words

see after rth row either r+1 th row is identical or
r+1 th row is partially identical with a lesser no of identical groups

i think the above statement directly solves the problem

11
Devil ·

Thanx for ur opinion, sir.

341
Hari Shankar ·

Your argument does not seem to use induction.

Also, there is no necessity that you will get numbers that repeat 1,2,3, times and so on. You could have several numbers all repeating the same number of times.

For induction to work, you should be able to ignore, say, one particular number k in the row. That means in that row k appears k times. But things get ruined if there is another number in the row that appears k times.

Induction is hence not an option.

11
Devil ·

I dunnow whether I'm correct or not....pls correct me if I'm wrong.... P(n) denote the statement that a row repeats at some point with n integers initially repeating.... P(1) is true, Let P(n) be true, For proving P(n+1), we have n+1 integers are repeating, so ai represent the number that repeats i times, i ranges from 1 to (n+1)... So a1,a2,a3,....repeat 1,2,3 times respy... Randomly any order like a1 a3 a2 a2 a3 a4 a3 ..... The next row contains 1 3 2 2 3 4 3 ...... Next row has 1 3 2 2 3 ........exactly identical like the last......

341
Hari Shankar ·

Solution Pending!

9
Celestine preetham ·

oops realised my mistake :)

ok anyway yes we can prove this easily

row 1
im shuffling the order and grouping like terms

a1(r1s) a2 (r2s) .......... an (rns)

where a1 + a2 + .........an= 1000

now we have in 2nd row

a1(a1s) a2(a2s) ...............an(ans)

if all ar's are distinct then 3rd row = 2nd row

oops gtg will finish this soon

341
Hari Shankar ·

would you like to go over the problem statement again?

9
Celestine preetham ·

wat is it so simple

the 2nd row itself is equal to 3rd row always !

Your Answer

Close [X]